Luận văn Một số lớp bài toán về dãy số

LỜI NÓI ĐẦU . . . . . . . . . . . . . . . . . . . . . . . . . . . . . . . . . . . . . . . . . . . . 4

Chương 1. Các kiến thức cơ bản . . . . . . . . . . . . . . . . . . . . . . . . . . . . . . . . . . . . . . 6

1.1. Dãy số . . . . . . . . . . . . . . . . . . . . . . . . . . . . . . . . . . . . . . . . . . . . . . . . . . . . . . . . . 6

1.1.1. Định nghĩa dãy số . . . . . . . . . . . . . . . . . . . . . . . . . . . . . . . . . . . . . . . . . . . . . . . . 6

1.1.2. Dãy số đơn điệu . . . . . . . . . . . . . . . . . . . . . . . . . . . . . . . . . . . . . . . . . . . . . . . . . . . 6

1.1.3. Dãy số bị chặn . . . . . . . . . . . . . . . . . . . . . . . . . . . . . . . . . . . . . . . . . . . . . . . . . . . . 7

1.1.4. Cấp số cộng, cấp số nhân . . . . . . . . . . . . . . . . . . . . . . . . . . . . . . . . . . . . . . . . . . 7

1.1.5. Các cách cho dãy số . . . . . . . . . . . . . . . . . . . . . . . . . . . . . . . . . . . . . . . . . . . . . . . 8

1.1.6. Dãy Fibonacci . . . . . . . . . . . . . . . . . . . . . . . . . . . . . . . . . . . . . . . . . . . . . . . . . . . 11

1.2. Giới hạn của dãy số. . . . . . . . . . . . . . . . . . . . . . . . . . . . . . . . . . . . . . . . . . . . 11

Chương 2. Một số lớp bài toán về dãy số . . . . . . . . . . . . . . . . . . . . . . . . . . . . . 15

2.1. Lớp bài toán có tính chất số học của dãy . . . . . . . . . . . . . . . . . . . . . . . . 15

2.2. Lớp các bài toán dãy số có bản chất đại số . . . . . . . . . . . . . . . . . . . . . . 23

2.3. Lớp các bài toán về bất đẳng thức dãy. . . . . . . . . . . . . . . . . . . . . . . . . . 27

2.4. Sử dụng lượng giác giải các bài toán về dãy. . . . . . . . . . . . . . . . . . . . . 46

2.5. Lớp các bài toán về giới hạn của dãy . . . . . . . . . . . . . . . . . . . . . . . . . . . 53

2.5.1. Phương pháp sử dụng định nghĩa tính giới hạn . . . . . . . . . . . . . . . . . . . . . . 53

2.5.2. Tính giới hạn nhờ sử dụng tính đơn điệu và bị chặn . . . . . . . . . . . . . . . . . . 54

2.5.3. Tính giới hạn nhờ sử dụng định lý hàm số co . . . . . . . . . . . . . . . . . . . . . . . . 57

2.5.4. Phương pháp sử dụng tổng tích phân tính giới hạn . . . . . . . . . . . . . . . . . . . 58

2.5.5. Tính giới hạn dựa vào việc giải phương trình sai phân . . . . . . . . . . . . . . . . 59

2.5.6. Sử dụng dãy phụ để tính giới hạn . . . . . . . . . . . . . . . . . . . . . . . . . . . . . . . . . . 60

pdf73 trang | Chia sẻ: mimhthuy20 | Lượt xem: 644 | Lượt tải: 0download
Bạn đang xem trước 20 trang tài liệu Luận văn Một số lớp bài toán về dãy số, để xem tài liệu hoàn chỉnh bạn click vào nút DOWNLOAD ở trên
iả sử rằng 2n số thực a1, · · ·,an,b1, · · ·,bn (n≥ 3) thỏa mãn các điều kiện sau: (a)a1 +a2 + · · ·+an = b1 +b2 + · · ·+bn; (b)0 < a1 = a2,ai +ai+1 = ai+2, i = 1,2, · · ·,n−2; (c)0 < b1 ≤ b2,bi +bi+1 ≤ bi+2, i = 1,2, · · ·,n−2. Chứng minh rằng:an−1 +an < bn−1 +bn. Lời giải Cho dãy Fn là dãy Fibonacci với F0 = 0,F1 = 1, sao cho: ai = Fi−1a1. Đặt d2 = b2−b1 và di = bi−bi−1−bi−2 với i > 2. Khi đó dễ thấy rằng: bi = Fi−1b1 +Fi−2d2 + · · ·+F1di. Ta có tính chất sau của dãy Fn: F0 + · · ·+Fk = Fk+2−1(có thể chứng minh lại bằng phương pháp quy nạp). Khi đó có: 27 bn−2 +bn b1 + · · ·+bn−2 = Fn+1b1 +Fnd2 + · · ·+F2dn (Fn−1)b1 +(Fn−1−1)d2 + · · ·+(F1−1)dn ≥ Fn+1b1 (Fn−1)b1 = an−1 +an a1 + · · ·+an−2 Bất đẳng thức đầu là hệ quả của kết quả sau: nếu a,b,c,d > 0 và ab ≤ c d thì a b ≤ a+ c b+d . Đặt s = a1 + · · ·+an; thì ta có: an−1 +an s−an−1−an ≤ bn−1 +bn s−bn−1−bn . Vì f (x) = x s− x là hàm tăng trên [0,s], do vậy ta có :an−1 +an ≤ bn−1 +bn. *Nhận xét: Lời giải trên đã sử dụng một tính chất của dãy Fibonacci- Một dãy đặc biệt đã được nhắc đến ở chương 1. Sau đây ta xét thêm một số tính chất khác của dãy Fibonacci: 1. un+2 = 1+u1 +u2 + ...+un 2.u1 +u3 +u5...+u2n−1 = u2n 3.u2 +u4 +u6...+u2n = u2n+1−1 4.u21 +u 2 2 + ...+u 2 n = un.un+1 5.u22n = u1u2 +u2u3 + ...+u2n−1.u2n 6.un+1un+2−unun+3 = (−1)n 7.u2n−un+1un−1 = (−1)n+1 Chứng minh các tính chất này khá đơn giản Tuy nhiên sự vận dụng các tính chất trên là rất lớn. Ví dụ như bài toán sau: Bài toán 2( Shortlist 1997 [8]) Với mọi số nguyên n ≥ 2 hãy xác định giá trị nhỏ nhất của tổng n ∑ i=0 ai có thể đạt được từ dãy số không âm a0,a1, · · ·,an thỏa mãn điều kiện a0 = 1,ai ≤ ai+1 + ai+2 với i = 0, · · ·,n−2. Lời giải 28 Ta dễ thấy rằng giá trị nhỏ nhất của n ∑ i=0 ai sẽ đạt được khi a0,a1, · · ·,an thỏa mãn hệ sau:  a1 +a2 = 1 a2 +a3 = a1 a3 +a4 = a2 · · · · · · · · · ak−1 +ak = ak−2 · · · · · · · · · an−1 +an = an−2 an = 0 Do vậy ai = fn−i fn , với fi với i = 0, · · ·,n là dãy Fibonacci : f0 = 0, f1 = 1, f2 = 1, · · · Và : Min n ∑ i=0 ai = 1 fn ( f0 + f1 + · · ·+ fn−1 + fn) = 1+ fn+1−1 fn . Bài toán 3(China 1997[10]) Cho x1,x2, · · ·,x1997là các số thực thỏa mãn các điều kiện sau: i,− 1√ 3 ≤ xn ≤ √ 3. ii, ∑xi =−318 √ 3. Tìm giá trị lớn nhất của ∑x12i . Lời giải Vì x12 là hàm lồi đối với x, ∑x12i đạt lớn nhất khi xi là điểm cuối của đoạn. Giả sử có n số xi bằng − 1√3 , 1996−n bằng √ 3 và phần tử cuối cùng bằng −318 √ 3+ n√ 3 − (1996−n) √ 3. 29 Số này sẽ được chọn sao cho thỏa mãn: −1√ 3 ≤−318 √ 3+ n√ 3 − (1996−n) √ 3≤ √ 3. Điều này tương đương với: −1≤ 4n− 6942 ≤ 3. Chỉ có số nguyên :n = 1736 thỏa mãn điều kiện này, và giá trị đó là: 2√ 3 , và giá trị lớn nhất là 1736.3−6 +260.36 +(43) 6. Bài toán 4(VMO-1999 [4]) Cho dãy (u)n được xác định bởi: u1 = 1,u2 = 2 và un+2 = 3un+1−un với n=1,2,3,... Chứng minh rằng: un+2 +un ≥ 2+ u2n+1 un , với mọi n=1,2,3,... Lời giải Dễ thấy u3 = 5 và dãy (un) (n=1,2,...) là dãy số dương tăng. Từ hệ thức un+2 = 3un+1−un ta có: (un+2 +un)un = 3un+1un = (un+1 +un−1)un+1 (n=2,3,...) Suy ra: un+2un−u2n+1 = un+1un−1−u2n (n=2,3,...) Như vậy: un+2un−u2n+1 = un+1un−1−u2n = .... = u3u1−u22 = 1. Từ đó ta có: un+2 + un = 1+u2n+1 un + un = 1 un + un + u2n+1 un . Hay un+2 + un ≥ 2+ u2n+1 un , với mọi n=1,2,3,... Bài toán 4(Bulgaria 1996 [10]) Cho dãy {an}∞n=1xác định bởi: a1 = 1;an+1 = an n + n an , n≥ 1. Chứng minh rằng với n≥ 4,⌊a2n⌋= n. Lời giải Điều cần chứng minh sẽ tương đương với việc ta chứng minh:√ n≤ an ≤ n√ n−1 , n > 1. Ta sẽ chứng minh bằng quy nạp: n = 1,2,3 khẳng định đúng. Giả sử đúng với n, ta cần chứng minh khẳng định đúng với n+1: Thật vậy, ta xét : fn(x) = x n + n x . Với n≥ 3, ta có: an+1 = fn(an)≥ fn( n√ n−1) = n√ n−1 > √ n+1 30 Mặt khác sử dụng: an > n−1√ n−2 ta có với n≥ 4: an+1 = fn(an) < fn( n−1√ n−2) = (n−1)+n2(n−2) (n−1)n√n−2 < √ n+2. Vậy ta có điều phải chứng minh. Bài toán 5(China 1997 [10]) Cho a1,a2, · · · là các số nguyên không âm thỏa mãn an+m ≤ an + am với m,n ∈ N. Chứng minh rằng: an ≤ ma1 +( n m −1)am. Lời giải Sử dụng quy nạp theo k, an ≤ kam + an−mk với k < m n . Đặt n = mk + r với r ∈ {1, · · ·,m}, khi đó: an ≤ kam +ar = n− r m am +ar ≤ n−m m am +ma1. do am ≤ a1 và ar ≤ ra1. Bài toán 6(Taiwan 1997 [10]). Cho n≥ 3 là số nguyên, và giả sử rằng dãy a1,a2, · · ·an thỏa mãn ai−1 +ai+1 = kiai với số nguyên dương ki. Chứng minh rằng 2n≤ ∑ki ≤ 3n. Lời giải Bất đẳng thức trái 2n ≤ k1 + k2 + · · ·+ kn ta sử dụng AM-GM để chứng minh với chú ý: k1 + k2 + · · ·+ kn = n ∑ i=1 ai ai+1 + ai+1 ai . Mặt khác, ta cũng chỉ ra được rằng k1 + k2 + · · ·+ kn ≤ 3n− 2 với n ≥ 2 bằng quy nạp theo n. Với n = 2, nếu a1 ≥ a2, thì 2a2 = k1a1, và ta có : hoặca1 = a2và k1 + k2 = 4 = 3.2−2,hoặc a1 = 2a2 và k1 + k2 = 4 = 3.2−2. Với n > 2, ta có thể giả sử ai là không đồng thời bằng sau, khi đó tồn tại i sao cho ai ≥ ai−1,ai+1 với bất đẳng thức ngặt ở ít nhất một trong hai trường hợp. Khi đó ai−1 +ai+1 < 2ai và ki = 1. Ta cũng có kết luận dãy khi bỏ đi ai vẫn thỏa mãn điều kiện đưa ra với ki−1và ki+1 giảm dần về 1 và bỏ đi ki. Do tổng kết quả ki lớn nhất là 3(n−1)−2 do giả thiết quy nạp. , vậy tổng ki ban đầu lớn nhất là 3n−2. 31 Bài toán 7(China 1998 [10]). Cho n là một số nguyên. Tồn tại hay không dãy nguyên dương a1,a2, · · ·,an,b1,b2, · · ·,bn sao cho:∑ai = ∑bi và n−1 > ∑ ai−bi ai +bi > n−1− 1 1998 . Lời giải Câu trả lời là có. Trước hết ta sẽ chứng minh: Nếu a1,a2, · · ·,an,b1,b2, · · ·,bn là 2n số nguyên dương phân biệt sao cho a1 +a2 + · · ·+an = b1 +b2 + · · ·+bn, khi đó: n−1 > n ∑ i=1 ai−bi ai +bi Thật vậy, từ điều kiện, ta thấy rằng tồn tại i1,1 ≤ i1 ≤ n, sao cho bi1 > ai1 . Do đó 2bi1 ai1 +bi1 > 1 và n ∑ i=1 ai−bi ai +bi = n ∑ i=1 ( 1− 2bi ai +bi ) = n− n ∑ i=1 2bi ai +bi < n−1. Cho N là một số nguyên dương. Với i = 1,2, · · ·,n− 1, đặt ai = N(2i− 1),bi = 2i. Từ điều kiện a1,a2, · · ·,an,b1,b2, · · ·,bn ta có: n ∑ i=1 ai =−(n−1)N +2N. (n−1)n2 +an = 2. (n−1)n 2 +bn = n ∑ i=1 bi. Do vậy bn = an + (n− 1)[N(n− 1)− n] với an đã được xác định. Từ khẳng định chứng minh lúc đầu, ta chỉ cần chứng minh bất đẳng thức trong điều kiện : n−1 > ∑ ai−bi ai +bi > n−1− 1 1998 . Nhưng điều đó lại suy ra từ: n−1 > n ∑ i=1 ai−bi ai +bi = n− 2an +2(n−1)[N(n−1)−n] 2an +(n−1)[N(n−1)−n] − n−1 ∑ i=1 4i 2i+N(2i−1) . Đại lượng này sẽ tiến đến n−1 khi N −→+∞ và an N −→+∞ (ví dụ, choan = N2). Do vậy với ε > 0 bất kỳ, ( ở đây ε = 1 1998 ), ta có thể tìm được N đủ lớn và an, sao 32 cho ai,bilà hoàn toàn phân biệt và n ∑ i=1 ai = n ∑ i=1 bi, và n−1 > n ∑ i=1 ai−bi ai +bi > n−1− ε . Bài toán 8(Iran 1998 [10]). Cho n1 < n2 < · · · là dãy các số tự nhiên sao cho với i < j, biểu diễn thập phân ni không xuất hiện trong số ngoài cùng bên trái của biểu diễn thập phân n j . Chứng minh rằng: ∞ ∑ i=1 1 ni ≤ 1+ 1 2 + · · ·+ 19 . Lời giải Rõ ràng ta chỉ cần với dãy hữu hạn là đủ. Giả sử cho một dãy hữu hạn, đặt M = 10N + d là phần tử lớn nhất của dãy, với 0≤ d ≤ 9. Khi đó N sẽ không thuộc dãy.Hơn nữa, bỏ đi 10N,10N +1, · · ·,10N +9 khỏi dãy nếu chúng xuất hiện và thêm N vào dãy khác sao cho tổng nghịch đảo là: ∑ i 1 ni + 1 N − 9 ∑ i=0 1 10N + i ≥∑i 1 ni . Do vậy ta bằng cách lặp lại việc thay thế và không làm giảm tổng nghịch đảo. Quá trình này là hữu hạn ( vì dãy là hữu hạn) và được dãy là {1, · · ·,9}, để dãy này có tổng các nghịch đảo là lớn nhất. Bài toán 9(Belarus 1999): Cho hai dãy số thực x1,x2, · · ·, và y1,y2, · · ·, được định nghĩa như sau: x1 = y1 = √ 3, xn+1 = xn + √ 1+ x2n, yn+1 = yn 1+ √ 1+ y2n Với mọi n≥ 1. Chứng minh rằng 2 1. Lời giải Đặt zn = 1 yn và chú ý rằng phương trình hồi quy yn tương đương với : zn+1 = zn + √ 1+ z2n. 33 Chú ý rằng z2 = √ 3 = x1; vì xi và zi thỏa mãn phương trình hồi quy giống nhau, điều này có nghĩa là zn = xn−1 với mọi n > 1. Do vậy, xnyn = xn zn = xn xn−1 . Do xi tăng, với n > 1 nên ta có: x2n−1 ≥ x21 = 3 > 1 3 , suy ra 2xn−1 > √ 1+ x2n−1 → 3xn−1 > xn. Ta lại có, √ 1+ x2n−1 > xn−1 → xn > 2xn−1. Hơn nữa, 2 < xnyn = xn xn−1 < 3. Ta có điều phải chứng minh. Bài toán 10(Romania 1999 [10]) Cho A = {a1,a2, · · ·} ⊂ N là tập thỏa mãn với mọi dãy con phân biệt B,C ⊆ A, ∑ x∈B x 6= ∑ x∈C x. Chứng minh rằng: 1 a1 + 1 a2 + · · ·+ 1 an < 2. Lời giải Trước hết ta chứng minh bổ đề sau: Bổ đề: Cho dãy x1,y1,x2,y2, · · ·,xn,yn là các số thực dương thỏa mãn: (i), x1y1 < x2y2 < · · ·< xnyn; (ii), x1 + x2 + · · ·+ xk ≥ y1 + y2 + · · ·+ yk với mọi k = 1,2, · · ·,n. Khi đó ta có : 1 x1 + 1 x2 + · · ·+ 1 xn ≤ 1 y1 + 1 y2 + · · ·+ 1 yn . Thật vậy ta có: Đặt pii = 1 xiyi ,δi = xi − yi với mọi 1 ≤ i ≤ n. Ta có thể giả sử pi1 > pi2 > · · ·> pin > 0 và k ∑ i=1 δi ≥ 0 với mọi 1≤ k ≤ n. Chú ý rằng: n ∑ k=1 ( 1 yk − 1 xk = n ∑ k=1 pikδk ) = pin n ∑ i=1 δi + n−1 ∑ k=1 (pik−pik+1)(δ1 +δ2 + · · ·+δk)≥ 0, 34 Vậy bổ đề được chứng minh. Quay lại bài toán, không mất tính tổng quát ta giả sử a1 < a2 < · · · < an, và đặt yi = 2i−1 với mọi i. Rõ ràng: a1y1 < a2y2 < · · ·< anyn. Với k bất kỳ, tổng 2k − 1 được tạo ra bằng cách chọn ít nhất một trong dãy rời rạc a1,a2, · · ·,ak . Do vậy lớn nhất trong đó là k ∑ i=1 ai, nhỏ nhất là 2k − 1. Do vậy với k = 1,2, · · ·,n ta có: a1 +a2 + · · ·+ak ≥ 2k−1 = y1 + y2 + · · ·+ yk. Áp dụng bổ đề, ta có: 1 a1 + 1 a2 + · · ·+ 1 an < 1 y1 + 1 y2 + · · ·+ 1 yn = 2− 1 2n−1 < 2. Đây là điều cần chứng minh. Bài toán 11(Bulgaria 1999 [8]). Chứng minh rằng với số nguyên bất kỳ n,n ≥ 3, tồn tại n số nguyên dương a1,a2, · · ·,an trong cấp số cộng, và n số nguyên dương b1,b2, · · ·,bn trong cấp số nhân, sao cho: b1 < a1 < b2 < a2 < · · ·< bn < an. Đưa ra ví dụ về 2 dãy này với mỗi dãy là ý nhất 5 số hạng. Lời giải Ta tìm dãy mà bn = an−1 +1 và bn−1 = an−2 +1. Viết d = an−1−an−2. Khi đó với 2≤ i, j ≤ n−1 ta có bi+1−bi ≤ bn−bn−1 = d, sao cho b j = bn + n−1 ∑ i= j (bi−bi+1) > an−1 +(n− j)d = a j−1. Và nếu ta chắc chắn b1 < a1, khi đó b j = b1 + j−1 ∑ i=1 (bi+1− bi)≤ a1 +( j− 1)d = a j với mọi j, nên chuỗi bất đẳng thức đã được thỏa mãn. Cho b1,b2, · · ·,bn bằng kn−1,kn−2(k + 1), · · ·,k0(k + 1)n−1, ở đó k là một giá trụ đã được xác định sau đó. Ta cũng đặt an−1 = bn − 1 và an−2 = bn−1 − 1, và định nghĩa ai khác theo đó. Khi đó d = an − an−1 = bn − bn−1 = (k + 1)n−2, và a1 = 35 (k +1)n−2(k +3−n)−1. Do vậy, ta chỉ cần lấy k sao cho: (k +1)n−2(k +3−n)−1− kn−1 > 0. Nhìn vế trái của đa thức ẩn k, hệ số của kn−1 bằng 0 nhưng hệ số của kn−2 bằng 1. Có nghĩa là biểu thức dương với k đủ lớn và ta có thể tìm được dãy a1,a2, · · ·,an và b1,b2, · · ·,bn thỏa mãn yêu cầu bài toán. Với n = 5, ta tìm k sao cho: (k +1)3(k−2)−1− k4 > 0. Ta thấy rằng k = 5 thỏa mãn và ta có: 625 < 647 < 750 < 863 < 900 < 1079 < 1080 < 1295 < 1296 < 1511. Bài toán 12(Romania 2000 [10]): Cho a là một số thực dương và {xn} (n≥ 1) là một dãy số thực sao cho x1 = a và xn+1 ≥ (n+2)xn− n−1 ∑ k=1 kxk, với mọi n≥ 1. Chứng minh rằng tồn tại số nguyên dương n sao cho xn > 1999!. Lời giải Ta sẽ chứng minh bằng quy nạp theo n≥ 1 rằng: xn+1 > n ∑ k=1 kxk > a.n! Với n = 1, ta có x2 ≥ 3x1 > x1 = a. Giả sử rằng khẳng định đúng đến n. Khi đó: xn+2 ≥ (n+3)xn+1− n ∑ k=1 kxk = (n+1)xn+1 +2xn+1− n ∑ k=1 kxk > (n+1)xn+1 +2 n ∑ k=1 kxk− n ∑ k=1 kxk = n+1 ∑ k=1 kxk, Hơn nữa, x1 > 0 theo định nghĩa và x2,x3, · · ·,xn cũng dương theo giả thiết quy nạp; do vậy xn+2 > (n+1)xn+1 > (n+1)(a.n!) = a.(n+1)!. Như vậy khẳng định được chứng minh hoàn toàn. Với n đủ lớn, ta có xn+1 > n!.a > 1999!. 36 Bài toán 13(APMO1999). Cho a1,a2, · · · là dãy các số thực thỏa mãn: ai+ j ≤ ai +a j với mọi i, j = 1,2, · · ·. Chứng minh rằng: a1 + a2 2 + a3 3 + · · ·+ an n ≥ an. với mọi số nguyên dương n. Lời giải Ta chứng minh bổ đề sau: Bổ đề: Nếu m,n là các số nguyên dương với m ≥ n, khi đó a1 + a2 + · · ·+ an ≥ n(n+1) 2m .am. Thật vậy, ta sẽ chứng minh kết quả cho m = n bằng cách cộng các bất đẳng thức : a1 + an−1 ≥ an,a2 + an−2 ≥ an, · · ·,an−1 + a1 ≥ an,2an ≥ 2an, và chia cho 2. Với số nguyên dương j, viết β j = a1 +a2 + · · ·+a j1+2+ · · ·+ j . Khi đó bất đẳng thức với m = n = j = k + 1 là tương đương với β j ≥ a jj và βk ≥ βk+1; nên khi m ≥ n ta có βn ≥ βn+1 ≥ · · · ≥ βm ≥ am m . Như vậy bổ đề được chứng minh. Từ bất đẳng thức đã chứng minh ta biểu diễn a1 + a2 2 + · · ·+ an n như tổng Abel và sau đó áp dụng bổ đề nhiều lần: a1 + a2 2 + · · ·+ an n = 1 n (a1 +a2 + · · ·+an)+ n−1 ∑ j=1 ( 1 j − 1 j +1 ) (a1 +a2 + · · ·+a j) ≥ 1 n . n(n+1) 2n an + n−1 ∑ j=1 1 j( j +1) . j( j +1) 2n an = an, Như vậy bài toán đã được chứng minh. Bài toán 14(Canada 2000 [8]). Giả sử các số thực a1,a2, · · ·,a100 thỏa mãn: (i), a1 ≥ a2 ≥ · · · ≥ a100 ≥ 0, (ii), a1 +a2 ≤ 100, (iii), a3 +a4 + · · ·+a100 ≤ 100. 37 Xác định giá trị lớn nhất có thể của a21 + a 2 2 + · · ·+ a2100, và tìm tất cả các dãy số dương a1,a2, · · ·,a100 đạt giá trị lớn nhất đó. Lời giải Với i ≥ 3, ta có : 0 ≤ ai ≤ a2 và do vậy ai(ai − a2) ≤ 0 dấu bằng xảy ra với ai ∈ {0,a2}. Cộng theo vế 98 bất đẳng thức này với nhau ta được : 100 ∑ i=3 ≤ a2. 100 ∑ i=3 ai. Do (iii) nên ta có :a2. 100 ∑ i=3 ai ≤ 100a2 dấu bằng xảy ra với 100 ∑ i=3 ai = 100 hoặc a2 = 0. Tương tự với (i) và (ii) ta có 0≤ a1 ≤ 100− a2. Do vậy, a21 ≤ (100− a2)2, với dấu bằng xảy ra khi a1 = 100−a2. Điều kiện (i) và (ii) kéo theo : 0≤ a2 ≤ 100−a1 ≤ 100−a2 hoặc 0≤ a2 ≤ 50. Do vậy, 2a2(a2−50)≥ 0 với dấu bằng xảy ra khi a2 = 0 hoặc a2 = 50. Hơn nữa, 100 ∑ i=1 a2i = a 2 1 +a 2 2 + 100 ∑ i=3 a2i ≤ (100−a2)2 +a22 +100a2 = 10000+2a2(a2−50)≤ 10000. Để bất đẳng thức xảy ra dấu bằng thì tất cả các dấu bằng ở trên đều phải xảy ra, điều đó có nghĩa là ta phải có: (a) {a3,a4, · · ·,a100 ⊆ {0,a2}; (b) 100 ∑ i=3 = 100 hoặc a2 = 0; (c)a1 = 100−a2; (d)a2 ∈ {0,50}. Những điều trên chỉ thỏa mãn khi dãy a1,a2, · · ·,a100 bằng : 100,0,0, · · ·,0 hoặc 50,50,50,50,0,0, · · ·,0. Hơn nữa các dãy này thỏa mãn điều kiện (i), (ii), (iii), và 100 ∑ i=1 a2i = 10000 với mỗi dãy. Hơn nữa, 10000 là tổng bình phương lớn nhất, và giá trị lớn nhất này chỉ đạt được với hai dãy trên. 38 Bài toán 15( Rusia 2000 [8]) Cho số nguyên lẻ a0 > 5, giả sử dãy a0,a1,a2, · · ·, ở đó: an+1 = a 2 n−5 nếu an lẻ an 2 nếu an chẵn với mọi n≥ 0. Chứng minh rằng dãy trên là không có biên. Lời giải Ta sẽ sử dụng quy nạp theo n để chỉ ra rằng a3n là lẻ và a3n > a3n−3 > · · ·> a0 > 5. Với n = 0 khẳng định đúng do giả thiết. Giả sử rằng khẳng định đúng với mọi n≤ k, ta cần chứng minh nó đúng với k +1. Vì a3k là lẻ, a23k ≡ 1(mod8) và do vậy a3k+1 = a23k − 5 ≡ 4(mod8). Do a3k+1 chia hết cho 4 nhưng không chia hết cho 8, nên a3(k+1) = a3k+1 4 là số lẻ. Hơn nữa, từ giả thiết quy nạp ta có a23k > 5a3k > 4a3k +5. Do đó, a3(k+1) = 1 4 (a23k− 5) > a3k. Đây chính là điều phải chứng minh. Bài toán 16(Shortlist 1988 [5]) Giả sử dãy với các số thực không âm thỏa mãn ak − 2ak+1 + ak+2 ≥ 0 và ∑ai ≤ 1 với k ≥ 1. Chứng minh rằng với k bất kỳ ta có 0≤ ak−ak+1 < 2k2 . Lời giải Từ giả thiết ta có :ak − ak+1 là đơn điệu giảm, đặc biệt nếuak − ak+1 = −δ với một số k nào đó và δ > 0. Khi đó am − am+1 ≤ −δ với mọi m > k, ak − am = (ak−ak+1)+(ak+1−ak+2)+ ···+(am−1−am)≤−(m−k)δ , và am ≥ ak +(m−k)δ với mọi m > k Đặc biệt với am > 1 với m đủ lớn, ta có mâu thuẫn. Do vậy ak−ak+1 ≥ 0 với mọi k. Giả sử với một số k nào đó, ta có ak−ak+1 ≥ 2k2 . Khi đó a j−a j+1 ≥ ak−ak+1 ≥ 2 k2 với mọi i ≤ j ≤ k và a j ≥ a j+1 + 2k2 ≥ a j+2 +2. 2 k2 ≥ · · · ≥ ak+1 +(k− j +1) 2 k2 . Đặc biệt, a1 + · · ·+ ak ≥ (1+ 2+ · · ·+ k) 2k2 + kak+1 > 1+ kak+1 ≥ 1. Mâu thuẫn với giả thiết. Vậy ta có điều phải chứng minh. Bài toán 17(IMO1995 [6]) 39 Tìm giá trị lớn nhất của x0 nếu tồn tại dãy x0,x1, · · ·,x1995 là dãy số thực dương với x0 = x1995, thỏa mãn: xi−1 + 2 xi−1 = 2xi + 1 xi với mọi i = 1, · · ·,1995. Lời giải Từ giả thiết ta có phương trình sau: 2x2i − ( xi−1 + 2 xi−1 ) xi +1 = 0 Từ đó ta có : xi = 1 xi−1 hoặc xi = 1 2 xi−1. Ta chỉ ra rằng: xi = 2kixεi0 với |ki| ≤ i và εi = (−1)ki+i. Điều này ta có thể chứng minh bằng quy nạp với xi = 1 xi−1 hoặc xi−1 2 . Nên ta có : x1995 = 2kxε0 với ε = ε1995,k = k1995,0≤ |k| ≤ 1995,ε = (−1)1995+k và x0 = x1995. Nếu k là số lẻ , thì ε = 1 do vậy 2k = 1 ( vô lý). Vậy giả sử k là số chẵn , thì ε =−1 và x20 = 2k, với |k| ≤ 1995 nên k ≤ 1994, do vậy x0 ≤ 2997. Cuối cùng ta chỉ ra x0 = 2997 là có thể xảy ra với x0 = 2997,xi = 1 2 xi−1 với 1 ≤ i ≤ 1994 và x1995 = 1 x1994 . Khi đó ta sẽ có : x0 = x1995 = 2997, do đó Maxx0 = 2997. Bài toán 18( IMO shortlist 1995 [5]) Giả sử rằng x1,x2,x3, · · · là các số thực dương thỏa mãn: xnn = n−1 ∑ i=0 xin với n = 1,2,3, · · ·. Chứng minh rằng với mọi n, ta có : 2− 1 2n−1 ≤ xn < 2− 12n . Lời giải Trước hết ta chứng minh với n≥ 2 là số tự nhiên, và u là một số thực không âm thỏa mãn: 1+u+u2 + · · ·+un−1 = un. 40 Khi đó ta có : 2− 1 2n−1 < u < 2− 1 2n . Thật vậy, nhân cả hai vế của : 1+u+u2 + · · ·+un−1 = un với 1−u, ta được: (1+u+u2 + · · ·+un−1)(1−u) = un(1−u), Hay 1− un = un − un+1, Tức là un+1 − 2un + 1 = 0. Do đó, u là nghiệm của : f (x) = xn+1−2xn +1. Ta chứng minh f (x) = xn+1−2xn +1 có nhiều nhất hai nghiệm không âm. Thật vậy, rõ ràng x = 1 là nghiệm, nhưng ta không quan tâm đến nghiệm này, bởi vì u 6= 1 ( do phương trình 1+ 1+ 12 + · · ·= 1n là không thể xảy ra với n≥ 2, ta có nghiệm này vì ta nhân phương trình với (1-u)). Điều này có nghĩa là ta cần chỉ ra một nghiệm u không âm thỏa mãn phương trình :1+u+u2 + · · ·+un−1 = un. Thật vậy, từ đạo hàm của f (x) là f (x)′ = (n + 1)xn− 2nxn−1, dễ thấy rằng f (x) là hàm đơn điệu giảm với x∈ [0; 2n n+1 ] và đơn điệu tăng với x∈ [ 2n n+1 ;+∞). Với mỗi đoạn trên , hàm f (x) chỉ có thể có một nghiệm, do vậy f (x) có nhiều nhất là hai nghiệm không âm. Như ta đã nói ở trên, x = 1 là một nghiệm. Ta quan tâm đến nghiệm còn lại. Ta cần chỉ ra nghiệm còn lại nằm trong khoảng (2− 1 2n−1 ;2− 1 2n ) bằng cách chứng minh f (2− 1 2n−1 ) < 0 và f (2− 1 2n ) > 0; vì f (x) là hàm liên tục, và khoảng trên không chứ một, do vậy chỉ ra khoảng trên chứa một nghiệm là bài toán được chứng minh. Do đó, vấn đề còn lại là chứng minh f (2− 1 2n−1 ) < 0 và f (2− 1 2n ) > 0. Trước hết, ta chỉ ra f (2− 1 2n−1 ) < 0. Thật vậy, Do f (2− 1 2n−1 ) = (2− 1 2n−1 )n+1−2(2− 1 2n−1 )n +1 = 1− (2− 1 2n−1 )n 1 2n−1 . Vậy chứng minh f (2− 1 2n−1 )< 0 tức là 1−(2− 1 2n−1 )n 1 2n−1 < 0 hay 1 < ( 2− 1 2n−1 )n 1 2n−1 . Ta viết lại thành : 2n−1 < ( 2− 1 2n−1 )n . Chia cả hai vế cho 2n ta được: 1 2 < ( 1− 1 2n ) . Bất đẳng thức này đúng theo bất đẳng thức Bernoulli (với − 1 2n > −1):( 1− 1 2n ) = ( 1+ ( − 1 2n ))n > 1+n ( − 1 2n ) = 1− n 2n = 1− 1 2 . n 2n−1 . 41 Với n≥ 2 có n≤ 2n−1 , nên n 2n−1 ≤ 1 và ( 1− 1 2n )n > 1− 1 2 . n 2n−1 ≥ 1− 1 2 = 1 2 . Ta chứng minh: f ( 2− 1 2n ) > 0 tương đương với( 2− 1 2n )n+1 −2 ( 2− 1 2n )n +1 > 0−→ 1− ( 2− 1 2n )n . 1 2n > 0−→ 1 > ( 2− 1 2n )n 1 2n −→ 2n > ( 2− 1 2n )n . Bất đẳng thức cuối cùng luôn đúng. Vậy bài toán đã được chứng minh. Bài toán 18(USAMO 1997 [9]) Cho dãy số nguyên không âm a1,a2, · · ·,a1997 thỏa mãn : ai +a j ≤ ai+ j ≤ ai +a j +1 với mọi i, j ≥ 1 và i + j ≤ 1997. Chứng minh rằng tồn tại một số thực x sao cho an = bnxc với 1≤ n≤ 1997. Lời giải Ta phải chỉ ra x ∈ [an n , an +1 n ]. Do vậy chỉ ra sự tồn tại của x với 1997 phần tử của dãy với các đoạn là rời nhau, tức là ∀m,n, an +1 n ≥ am m . Ta chứng minh bằng quy nạp theo m+n. Trường hợp m+n = 2 hay m = n = 1 bất đẳng thức là hiển nhiên. Nếu m = n bất đẳng thức hiên nhiên đúng. Nếu n > m, ta sử dụng thuật toán n = mq + r, với r < n. Do đó m + r < n + m và ta sử dụng giả thiết quy nạp ar +1 r ≥ am m −→ amq+r + 1 ≥ qam + ar + 1 = qam + r. ar +1 r ≥ qam + r am m = n am m −→ an +1 n ≥ am m . Nếu n < m, ta sử dụng thuật toán m = nq+ r với r < m, ta có r + n < m + n, ta lại sử dụng giả thiết quy nạp: ar r ≤ an +1 n −→ nam = naqn+r ≤ n(qan + ar + q) ≤ nqan + nq + nr ar r ≤ nqan + nq+nr an +1 n = nqan + nq+ ran + r = m(an + 1)−→ am m ≥ an +1 n . Vậy bài toán đã được chứng minh. Bài toán 19(Shortlist 1989 [5]) Cho tập các số thực {a0,a1, · · ·,an} thỏa mãn các điều kiện sau: 42 (i) a0 = an = 0, (ii) Với 1≤ k ≤ n−1, ak = c+ n−1 ∑ i=k ai−k.(ai +ai+1) Chứng minh rằng :c ≤ 1 4n . Lời giải Đặt Sk = k ∑ i=0 ai(k = 0,1, · · ·,n). Khi đó: Sn = n−1 ∑ k=0 ak = nc+ n−1 ∑ k=0 n−1 ∑ i=k ai−k(ai +ai+1) = nc+ n−1 ∑ i=0 i ∑ k=0 ai−k(ai +ai+1) = nc+ n−1 ∑ i=0 (ai +ai+1) i ∑ k=0 ai−k = nc+ n−1 ∑ i=0 (ai +ai+1) i ∑ t=0 ai , với t = i− k = nc+ n−1 ∑ i=0 (ai +ai+1)S1 = nc+(S1S0 +(S2−S0)S1 +(S3−S1)S2 + · · ·+(Sn−Sn−2)Sn−1) = nc+S2n (vì Sn−1 = Sn). Do vậy ta có :S2n−Sn +nc = 0 Vì Sn là số thực nên: 1≥ 4nc. Ta có điều phải chứng minh. Bài toán 20( Shortlist 1994 [5]) Cho a0 = 1994 và an+1 = a2n an +1 với n≥ 0. Chứng minh rằng banc= 1994−n với 0≤ n≤ 998. Lời giải Ta có an−an+1 = an− a 2 n an +1 = 1− 1 an +1 > 0. Do vậy, a0 > a1 > · · ·> an > · · · Lại có :an = a0 +(a1−a0)+ ···+(an−an−1)= 1994−n+ 1 a0 +1 + ···+ 1 an−1 +1 > 1994−n. Với 1≤ n≤ 998, ta có : 1 a0 +1 + · · ·+ 1 an−1 +1 < n an−1 +1 < 998 a997 +1 < 998 1994−997+1 = 1 Do vậy banc= 1994−n. Bài toán 21(Shortlist 1996) 43 Cho a > 2. Ta định nghĩa như sau: a0 = 1,a1 = 1,an+1 = ( a2n a2n−1 −2 ) an Chứng minh rằng với mọi k ∈N ta có 1 a0 + · · ·+ 1 ak ≤ 1 2 ( 2+a− √ a2−4 ) Lời giải Từ a > 2 ta có thể viết a = b + 1b với b là một số thực dương. Khi đó ta có : a2−2 = b2 + 1b2 và: a2 = (a 2−2)a = ( b2 + 1b2 )( b+ 1b ) a3 = (( a2 a1 )2 −2 ) a2 = (( b2 + 1b2 )2 −2 ) a2 = ( b4 + 1b4 )( b2 + 1b2 )( b+ 1b ) . Tiếp tục quá trình trên ta thu được: an = ( b2n−1 + 1 b2n−1 ) · · · ( b2 + 1b2 )( b+ 1b ) . Do vậy, n ∑ i=1 1 ai = 1+ bb2 +1 + b3 (b2 +1)(b4 +1) + · · ·+ b2n−1 (b2 +1)(b4 +1) · · · (b2n +1) Ta lại có : 1 2 (a+2− √ a2−4) = 1 2 ( b+ 1b +2− ( b− 1b )) = 1+ 1b . Do vậy ta phải chứng minh với mọi b > 0 thì, b2 1+b2 + b4 (1+b2)(1+b4) + · · ·+ b2n (1+b2)(1+b4) · · · (1+b2n) < 1. 44 Mà với mọi số thực dương a1,a2, · · ·,an, n ∑ j=1 a j (1+a1) · · · (1+a j) = 1− 1 (1+a1)(1+a2) · · · (1+an) Do vậy : n ∑ j=1 b2 j (1+b2) · · · (1+b2 j ) = 1− 1 (1+b2) · · · (1+b2n) < 1. Vậy bài toán đã được chứng minh. Bài toán 22( VMO1997 [4]) Tìm số thực α lớn nhất để tồn tại dãy vô hạn a1,a2, · · · các số nguyên dương sao cho tính chất sau được thỏa mãn: (a) Với mỗi n ∈N,an > 1997n (b)Với mọi n≥ 2,aαn không vượt quá ước chung lớn nhất của tập {ai +a j : i+ j = n}. Lời giải Giá trị lớn nhất có thể của α là 1 2 Trước hết, giả sử (an)∞n=1 là một dãy sao cho thỏa mãn điều kiện (a) và (b). Ta có kết quả sau: Với mọi ε > 0, tồn tại vô hạn giá trị n ∈ N mà :a2n ≥ a2−εn Thật vậy, Với ε > 0, và giả sử tồn tại N ∈N sao cho mọi n > N,a2n < a2−εn . Logarit hóa hai vế và chia cho 2n ta được : loga2n 2n < 2− ε 2 . logan n nên a2kn 2kn < ( 2− ε 2 )k logan n −> 0 khi k−> ∞ điều này không thể xảy ra với an ≥ 1997n nên logan n ≥ log1997,∀n. Kết quả được chứng minh. Giả sử n là một trong những giá trị cho bởi kết quả trên, sao cho: a2−εn ≤ a2n. Khi đó a (2−ε)α n ≤ aα2n ≤UCLN{ai +a j|i+ j = 2n} ≤ 2an. 45 nên 2≥ a1−(2−ε)αn ≥ 1997n(1−(2−ε)α); bởi vì điều này đúng với vô hạn giá trị n ∈N nên ta phải có α ≤ 1 2− ε . Từ ε > 0 bé tùy ý, nến ta có α ≤ 1 2 . Ta sẽ đưa ra một dãy thỏa mãn điều kiện (a) và (b) với α = 1 2 . Ký hiệu Fn là phần tử thứ n trong dãy Fibonacci. Cho t là số nguyên chẵn sao cho F2tn > 1997n,∀n ∈N và định nghĩa dãy (an)∞n=1 bởi: an = 3F2tn. Khi đó điều kiện (a) rõ ràng được thỏa mãn. Ta cần chỉ ra Ftn|F2ti +F2t j khi i+ j = n, để UCLN{ai +a j|i+ j = n} ≥ 3Ftn. Thật vậy: F2ti = Ft(i+ j)Ft(i− j)+1 +Ft(i+ j)−1Ft(i− j) F2t j = Ft(i+ j)Ft( j−i)+1 +Ft(i+ j)+1Ft( j−1) Do vậy F2ti +F2t j = 2Ft(i+ j)Ft(i− j)+1 + ( Ft(i+ j)+1−Ft(i+ j)−1 ) Ft( j−i) = Ft(i+ j) ( 2Ft(i− j)+1−Ft(i− j) ) Ta có : an = 3F2tn = 3Ft

Các file đính kèm theo tài liệu này:

  • pdfluanvanthacsi_chuaphanloai_358_9532_1870226.pdf
Tài liệu liên quan